I'm differentiating this wrong!












0












$begingroup$


I am self-teaching calculus and have been looking at the related rates practice problems here: https://www.whitman.edu/mathematics/calculus_online/section06.02.html



I am having trouble with the last problem, which relates to the rate at which a piece of paper will be cut by a pair of scissors. We are given the rate $dot{theta}$ at which the angle of a pair of open scissors ($theta$) changes with respect to time ($t$) and also some other numbers based on the diagram provided and a fixed point when the scissors are closing.



I'm happy that $x = (alpha sin beta)cdot sin(beta+theta)^{-1}$ as stated in the provided solution, which I arrived at by the law of sines. However, when I implicitly differentiate this and plug in the provided values, my answer is way off.



The task is to find the derivative of $x$ with respect to time, $dot{x}$. $alpha$ and $beta$ are constants, 20cm and 5 degrees respectively. $theta$ varies, but we know that its derivative with respect to time, $dot{theta}$, is -50 degrees per second.



My approach to differentiation has been to start off with the angle addition identity for $sin(a+b)$ to say that



$$dot{x} = (alpha sinbeta) cdot frac{d}{dt} sin(beta+theta)^{-1} = (alpha sinbeta) cdot frac{d}{dt} (sin(theta)cos(beta)+cos(theta)sin(beta))^{-1}$$



I'm then using the chain rule to find $frac{d}{dt}(sin(theta)cos(beta)+cos(theta)sin(beta))^{-1}$, and I'm ending up with:



$$-1cdot(sin(theta)cos(beta)+cos(theta)sin(beta))^{-2}cdot(cos(beta)cos(theta)-sin(theta)sin(beta))cdotdot{theta}$$



I then simplify this a bit, but I suspect I have done it wrong as plugging in the provided values doesn't get me to the right answer (~$3.79$cm/s)!



Can you see what I'm doing wrong? Any help is greatly appreciated.










share|cite|improve this question











$endgroup$












  • $begingroup$
    Does $sin(beta + theta)^{-1}$ means $sinfrac{1}{beta + theta}$ or $frac{1}{sin(beta + theta)}$?
    $endgroup$
    – Alex Vong
    Dec 22 '18 at 21:48












  • $begingroup$
    From the link it seems to be the latter, $$frac{1}{sin(beta+theta)}$$
    $endgroup$
    – mark
    Dec 22 '18 at 22:17












  • $begingroup$
    Indeed - the latter! Any ideas what's going on? Thanks for your help.
    $endgroup$
    – Nicholas James Bailey
    Dec 22 '18 at 23:44
















0












$begingroup$


I am self-teaching calculus and have been looking at the related rates practice problems here: https://www.whitman.edu/mathematics/calculus_online/section06.02.html



I am having trouble with the last problem, which relates to the rate at which a piece of paper will be cut by a pair of scissors. We are given the rate $dot{theta}$ at which the angle of a pair of open scissors ($theta$) changes with respect to time ($t$) and also some other numbers based on the diagram provided and a fixed point when the scissors are closing.



I'm happy that $x = (alpha sin beta)cdot sin(beta+theta)^{-1}$ as stated in the provided solution, which I arrived at by the law of sines. However, when I implicitly differentiate this and plug in the provided values, my answer is way off.



The task is to find the derivative of $x$ with respect to time, $dot{x}$. $alpha$ and $beta$ are constants, 20cm and 5 degrees respectively. $theta$ varies, but we know that its derivative with respect to time, $dot{theta}$, is -50 degrees per second.



My approach to differentiation has been to start off with the angle addition identity for $sin(a+b)$ to say that



$$dot{x} = (alpha sinbeta) cdot frac{d}{dt} sin(beta+theta)^{-1} = (alpha sinbeta) cdot frac{d}{dt} (sin(theta)cos(beta)+cos(theta)sin(beta))^{-1}$$



I'm then using the chain rule to find $frac{d}{dt}(sin(theta)cos(beta)+cos(theta)sin(beta))^{-1}$, and I'm ending up with:



$$-1cdot(sin(theta)cos(beta)+cos(theta)sin(beta))^{-2}cdot(cos(beta)cos(theta)-sin(theta)sin(beta))cdotdot{theta}$$



I then simplify this a bit, but I suspect I have done it wrong as plugging in the provided values doesn't get me to the right answer (~$3.79$cm/s)!



Can you see what I'm doing wrong? Any help is greatly appreciated.










share|cite|improve this question











$endgroup$












  • $begingroup$
    Does $sin(beta + theta)^{-1}$ means $sinfrac{1}{beta + theta}$ or $frac{1}{sin(beta + theta)}$?
    $endgroup$
    – Alex Vong
    Dec 22 '18 at 21:48












  • $begingroup$
    From the link it seems to be the latter, $$frac{1}{sin(beta+theta)}$$
    $endgroup$
    – mark
    Dec 22 '18 at 22:17












  • $begingroup$
    Indeed - the latter! Any ideas what's going on? Thanks for your help.
    $endgroup$
    – Nicholas James Bailey
    Dec 22 '18 at 23:44














0












0








0





$begingroup$


I am self-teaching calculus and have been looking at the related rates practice problems here: https://www.whitman.edu/mathematics/calculus_online/section06.02.html



I am having trouble with the last problem, which relates to the rate at which a piece of paper will be cut by a pair of scissors. We are given the rate $dot{theta}$ at which the angle of a pair of open scissors ($theta$) changes with respect to time ($t$) and also some other numbers based on the diagram provided and a fixed point when the scissors are closing.



I'm happy that $x = (alpha sin beta)cdot sin(beta+theta)^{-1}$ as stated in the provided solution, which I arrived at by the law of sines. However, when I implicitly differentiate this and plug in the provided values, my answer is way off.



The task is to find the derivative of $x$ with respect to time, $dot{x}$. $alpha$ and $beta$ are constants, 20cm and 5 degrees respectively. $theta$ varies, but we know that its derivative with respect to time, $dot{theta}$, is -50 degrees per second.



My approach to differentiation has been to start off with the angle addition identity for $sin(a+b)$ to say that



$$dot{x} = (alpha sinbeta) cdot frac{d}{dt} sin(beta+theta)^{-1} = (alpha sinbeta) cdot frac{d}{dt} (sin(theta)cos(beta)+cos(theta)sin(beta))^{-1}$$



I'm then using the chain rule to find $frac{d}{dt}(sin(theta)cos(beta)+cos(theta)sin(beta))^{-1}$, and I'm ending up with:



$$-1cdot(sin(theta)cos(beta)+cos(theta)sin(beta))^{-2}cdot(cos(beta)cos(theta)-sin(theta)sin(beta))cdotdot{theta}$$



I then simplify this a bit, but I suspect I have done it wrong as plugging in the provided values doesn't get me to the right answer (~$3.79$cm/s)!



Can you see what I'm doing wrong? Any help is greatly appreciated.










share|cite|improve this question











$endgroup$




I am self-teaching calculus and have been looking at the related rates practice problems here: https://www.whitman.edu/mathematics/calculus_online/section06.02.html



I am having trouble with the last problem, which relates to the rate at which a piece of paper will be cut by a pair of scissors. We are given the rate $dot{theta}$ at which the angle of a pair of open scissors ($theta$) changes with respect to time ($t$) and also some other numbers based on the diagram provided and a fixed point when the scissors are closing.



I'm happy that $x = (alpha sin beta)cdot sin(beta+theta)^{-1}$ as stated in the provided solution, which I arrived at by the law of sines. However, when I implicitly differentiate this and plug in the provided values, my answer is way off.



The task is to find the derivative of $x$ with respect to time, $dot{x}$. $alpha$ and $beta$ are constants, 20cm and 5 degrees respectively. $theta$ varies, but we know that its derivative with respect to time, $dot{theta}$, is -50 degrees per second.



My approach to differentiation has been to start off with the angle addition identity for $sin(a+b)$ to say that



$$dot{x} = (alpha sinbeta) cdot frac{d}{dt} sin(beta+theta)^{-1} = (alpha sinbeta) cdot frac{d}{dt} (sin(theta)cos(beta)+cos(theta)sin(beta))^{-1}$$



I'm then using the chain rule to find $frac{d}{dt}(sin(theta)cos(beta)+cos(theta)sin(beta))^{-1}$, and I'm ending up with:



$$-1cdot(sin(theta)cos(beta)+cos(theta)sin(beta))^{-2}cdot(cos(beta)cos(theta)-sin(theta)sin(beta))cdotdot{theta}$$



I then simplify this a bit, but I suspect I have done it wrong as plugging in the provided values doesn't get me to the right answer (~$3.79$cm/s)!



Can you see what I'm doing wrong? Any help is greatly appreciated.







implicit-differentiation






share|cite|improve this question















share|cite|improve this question













share|cite|improve this question




share|cite|improve this question








edited Dec 22 '18 at 21:05









postmortes

2,07531120




2,07531120










asked Dec 22 '18 at 11:52









Nicholas James BaileyNicholas James Bailey

1483




1483












  • $begingroup$
    Does $sin(beta + theta)^{-1}$ means $sinfrac{1}{beta + theta}$ or $frac{1}{sin(beta + theta)}$?
    $endgroup$
    – Alex Vong
    Dec 22 '18 at 21:48












  • $begingroup$
    From the link it seems to be the latter, $$frac{1}{sin(beta+theta)}$$
    $endgroup$
    – mark
    Dec 22 '18 at 22:17












  • $begingroup$
    Indeed - the latter! Any ideas what's going on? Thanks for your help.
    $endgroup$
    – Nicholas James Bailey
    Dec 22 '18 at 23:44


















  • $begingroup$
    Does $sin(beta + theta)^{-1}$ means $sinfrac{1}{beta + theta}$ or $frac{1}{sin(beta + theta)}$?
    $endgroup$
    – Alex Vong
    Dec 22 '18 at 21:48












  • $begingroup$
    From the link it seems to be the latter, $$frac{1}{sin(beta+theta)}$$
    $endgroup$
    – mark
    Dec 22 '18 at 22:17












  • $begingroup$
    Indeed - the latter! Any ideas what's going on? Thanks for your help.
    $endgroup$
    – Nicholas James Bailey
    Dec 22 '18 at 23:44
















$begingroup$
Does $sin(beta + theta)^{-1}$ means $sinfrac{1}{beta + theta}$ or $frac{1}{sin(beta + theta)}$?
$endgroup$
– Alex Vong
Dec 22 '18 at 21:48






$begingroup$
Does $sin(beta + theta)^{-1}$ means $sinfrac{1}{beta + theta}$ or $frac{1}{sin(beta + theta)}$?
$endgroup$
– Alex Vong
Dec 22 '18 at 21:48














$begingroup$
From the link it seems to be the latter, $$frac{1}{sin(beta+theta)}$$
$endgroup$
– mark
Dec 22 '18 at 22:17






$begingroup$
From the link it seems to be the latter, $$frac{1}{sin(beta+theta)}$$
$endgroup$
– mark
Dec 22 '18 at 22:17














$begingroup$
Indeed - the latter! Any ideas what's going on? Thanks for your help.
$endgroup$
– Nicholas James Bailey
Dec 22 '18 at 23:44




$begingroup$
Indeed - the latter! Any ideas what's going on? Thanks for your help.
$endgroup$
– Nicholas James Bailey
Dec 22 '18 at 23:44










0






active

oldest

votes











Your Answer





StackExchange.ifUsing("editor", function () {
return StackExchange.using("mathjaxEditing", function () {
StackExchange.MarkdownEditor.creationCallbacks.add(function (editor, postfix) {
StackExchange.mathjaxEditing.prepareWmdForMathJax(editor, postfix, [["$", "$"], ["\\(","\\)"]]);
});
});
}, "mathjax-editing");

StackExchange.ready(function() {
var channelOptions = {
tags: "".split(" "),
id: "69"
};
initTagRenderer("".split(" "), "".split(" "), channelOptions);

StackExchange.using("externalEditor", function() {
// Have to fire editor after snippets, if snippets enabled
if (StackExchange.settings.snippets.snippetsEnabled) {
StackExchange.using("snippets", function() {
createEditor();
});
}
else {
createEditor();
}
});

function createEditor() {
StackExchange.prepareEditor({
heartbeatType: 'answer',
autoActivateHeartbeat: false,
convertImagesToLinks: true,
noModals: true,
showLowRepImageUploadWarning: true,
reputationToPostImages: 10,
bindNavPrevention: true,
postfix: "",
imageUploader: {
brandingHtml: "Powered by u003ca class="icon-imgur-white" href="https://imgur.com/"u003eu003c/au003e",
contentPolicyHtml: "User contributions licensed under u003ca href="https://creativecommons.org/licenses/by-sa/3.0/"u003ecc by-sa 3.0 with attribution requiredu003c/au003e u003ca href="https://stackoverflow.com/legal/content-policy"u003e(content policy)u003c/au003e",
allowUrls: true
},
noCode: true, onDemand: true,
discardSelector: ".discard-answer"
,immediatelyShowMarkdownHelp:true
});


}
});














draft saved

draft discarded


















StackExchange.ready(
function () {
StackExchange.openid.initPostLogin('.new-post-login', 'https%3a%2f%2fmath.stackexchange.com%2fquestions%2f3049361%2fim-differentiating-this-wrong%23new-answer', 'question_page');
}
);

Post as a guest















Required, but never shown

























0






active

oldest

votes








0






active

oldest

votes









active

oldest

votes






active

oldest

votes
















draft saved

draft discarded




















































Thanks for contributing an answer to Mathematics Stack Exchange!


  • Please be sure to answer the question. Provide details and share your research!

But avoid



  • Asking for help, clarification, or responding to other answers.

  • Making statements based on opinion; back them up with references or personal experience.


Use MathJax to format equations. MathJax reference.


To learn more, see our tips on writing great answers.




draft saved


draft discarded














StackExchange.ready(
function () {
StackExchange.openid.initPostLogin('.new-post-login', 'https%3a%2f%2fmath.stackexchange.com%2fquestions%2f3049361%2fim-differentiating-this-wrong%23new-answer', 'question_page');
}
);

Post as a guest















Required, but never shown





















































Required, but never shown














Required, but never shown












Required, but never shown







Required, but never shown

































Required, but never shown














Required, but never shown












Required, but never shown







Required, but never shown







Popular posts from this blog

To store a contact into the json file from server.js file using a class in NodeJS

Redirect URL with Chrome Remote Debugging Android Devices

Dieringhausen